1987 AHSME Problems/Problem 7

Revision as of 08:36, 23 October 2014 by Timneh (talk | contribs) (Created page with "==Problem== If <math>a-1=b+2=c-3=d+4</math>, which of the four quantities <math>a,b,c,d</math> is the largest? <math>\textbf{(A)}\ a \qquad \textbf{(B)}\ b \qquad \textbf{(C)}...")
(diff) ← Older revision | Latest revision (diff) | Newer revision → (diff)

Problem

If $a-1=b+2=c-3=d+4$, which of the four quantities $a,b,c,d$ is the largest?

$\textbf{(A)}\ a \qquad \textbf{(B)}\ b \qquad \textbf{(C)}\ c \qquad \textbf{(D)}\ d \qquad \textbf{(E)}\ \text{no one is always largest}$


See also

1987 AHSME (ProblemsAnswer KeyResources)
Preceded by
Problem 6
Followed by
Problem 8
1 2 3 4 5 6 7 8 9 10 11 12 13 14 15 16 17 18 19 20 21 22 23 24 25 26 27 28 29 30
All AHSME Problems and Solutions

The problems on this page are copyrighted by the Mathematical Association of America's American Mathematics Competitions. AMC logo.png